Physics, asked by ashishrajput8404, 4 months ago

*If a gymnast sitting on a rotating stool with his arms outstretched suddenly lowers his arms:*

1️⃣ The angular velocity decreases
2️⃣ His moment of inertia decreases
3️⃣ The angular velocity remains constant
4️⃣ The angular momentum increases​

Answers

Answered by Anonymous
3

Answer:

As the gymnast lowers his hands, the radius of gyration decreases and hence moment of inertia also decreases. Angular momentum remains conserved and hence angular velocity increases.

the answer is option D

Similar questions